2017 AMC 12A Problems/Problem 6

Revision as of 15:40, 8 February 2017 by Speck (talk | contribs) (Created page with "Joy has <math>30</math> thin rods, one each of every integer length from <math>1 \text{ cm}</math> through <math>30 \text{ cm}</math>. She places the rods with lengths <math>3...")
(diff) ← Older revision | Latest revision (diff) | Newer revision → (diff)

Joy has $30$ thin rods, one each of every integer length from $1 \text{ cm}$ through $30 \text{ cm}$. She places the rods with lengths $3 \text{ cm}$, $7 \text{ cm}$, and $15 \text{cm}$ on a table. She then wants to choose a fourth rod that she can put with these three to form a quadrilateral with positive area. How many of the remaining rods can she choose as the fourth rod?


$\textbf{(A)}\ 16 \qquad\textbf{(B)}\ 17 \qquad\textbf{(C)}\ 18 \qquad\textbf{(D)}\ 19  \qquad\textbf{(E)}\ 20$